You are on page 1of 43

21EC 1203 DESIGN OF BASIC ELECTRONIC CIRCUITS (CO 2)

METHODS OF ANALYSING CIRCUITS

Basically there are two methods of analysing electrical circuits. They are mesh
analysis and node analysis.

2.1 MESH ANALYSIS

A loop is a closed path drawn starting at a node and tracing the path such that we
return to the original node without passing an intermediate node more than once. A
mesh is a special case of a loop. A mesh is defined as a loop which does not contain
any other loops within it. The network shown in Figure 2.1(a) has 3 meshes (A-B-D-
A, B-C-D-B, A-C-B-A) and they are indentified as 𝑀1 , 𝑀2 and 𝑀3 . A-B-C-D-A and A-
C-D-A are loops but not meshes because loop A-B-C-D-A contains loopsA-B-D-A
and B-C-D-B and loop A-C-D-A contains loops A-C-B-A, A-B-D-A and B-C-D-B.

R4 R4
I3
M3
R2 R3
A B C A B C
R3
R2

R1 R5 R1 R5

M1 M2 I1 I2
+ + R6 + + R6
V1 -
V2 - V1 -
V2 -

D D
(a) (b)

Figure 2.1(a) A 3 loop network (b) Network with loop currents marked

To apply mesh analysis, our first step is to check whether the circuit is planar
or not and the second is to select the mesh currents. The third step is to write the
KVL equations in terms of unknowns and solving them to find the final solution.

The current flowing in a mesh is defined as mesh current. As a matter of


convention, the mesh currents are assumed to flow in a mesh in the clockwise
direction. While employing KVL around each mesh, we travel around each mesh in
the clockwise direction and sum the voltage rises and drops encountered in that
particular mesh. The general convention is voltage drops are taken as positive and
voltage rises are taken as negative. We adopt this throughout.

1
Steps to be followed in mesh analysis.

1. Identify the meshes, assign an unknown current in each mesh and assign a
direction to it.

2. Apply KVL around the mesh and use ohm’s law to express the branch voltages in
terms of unknown mesh currents and the resistances.

2. Solve the simultaneous equations for unknown mesh currents.

The number of meshes = Branches – (Nodes − 1) . In Figure 2.1(a), the


number of mesh currents would be 8 − (6 − 1) = 3. In general, if a circuit has B
number of branches and N number of nodes including the reference node, then the
number of linearly independent mesh equations 𝑀 = 𝐵 − (𝑁 − 1).

Consider the 3 loop circuit shown in Figure 2.1(b). Let the loop currents be
𝐼1 , 𝐼2 and 𝐼3 respectively. The KVL equations are:

Loop1, (𝐴 − 𝐵 − 𝐷 − 𝐴), i. e. 𝑅2 − 𝑅5 − 𝑉2 − 𝑉1 − 𝑅1

In this, (𝐼1 − 𝐼3 ) is passing through 𝑅2 , (𝐼1 − 𝐼2 ) is passing through 𝑅5 and 𝐼1 is


passing through 𝑅1 . Also there are two voltage sources.
Voltage drop across 𝑅2 = (𝐼1 − 𝐼3 )𝑅2 ∶ positive

Voltage drop across 𝑅5 = (𝐼1 − 𝐼2 )𝑅5 ∶ positive

Voltage drop across source 𝑉2 = 𝑉2 ∶ positive

Voltage rise across source 𝑉1 = −𝑉1 : negative

Voltage drop across 𝑅1 = 𝐼1 𝑅1 ∶ positive

So the KVL equation for mesh 1 is

(𝐼1 − 𝐼3 )𝑅2 + (𝐼1 − 𝐼2 )𝑅5 + 𝑉2 − 𝑉1 + 𝐼1 𝑅1 = 0

or (𝑅1 + 𝑅2 + 𝑅5 )𝐼1 − 𝑅5 𝐼2 − 𝑅2 𝐼3 = 𝑉1 − 𝑉2

Loop2, (𝐵 − 𝐶 − 𝐷 − 𝐵), i. e. 𝑅3 − 𝑅6 − 𝑉2 − 𝑅5

In this, (𝐼2 − 𝐼3 ) is passing through 𝑅3 , 𝐼2 is passing through 𝑅6 and (𝐼2 − 𝐼1 ) is


passing through 𝑅5 . Also there is a voltage source 𝑉2.

Voltage drop across 𝑅3 = (𝐼2 − 𝐼3 )𝑅3 ∶ positive

Voltage drop across 𝑅6 = 𝐼2 𝑅6 ∶ positive

Voltage rise across 𝑉2 = −𝑉2 ∶ negative

Voltage drop across 𝑅5 = (𝐼2 − 𝐼1 )𝑅5 = positive

2
So the KVL equation for mesh 2 is

(𝐼2 − 𝐼3 )𝑅3 + 𝐼2 𝑅6 − 𝑉2 + (𝐼2 − 𝐼1 )𝑅5 = 0

−𝑅5 𝐼1 + (𝑅3 + 𝑅5 + 𝑅6 )𝐼2 − 𝑅3 𝐼3 − 𝑉2 = 0

The KVL equation around loop 3 is

𝑅4 𝐼3 + 𝑅3 (𝐼3 − 𝐼2 ) + 𝑅2 (𝐼3 − 𝐼1 ) = 0

−𝑅2 𝐼1 − 𝑅3 𝐼2 + (𝑅2 + 𝑅3 + 𝑅4 )𝐼3 = 0

By solving the above equations, we can find the currents 𝐼1 , 𝐼2 and 𝐼3 .

2.2 MESH EQUATIONS BY INSPECTION METHOD

The mesh equations for a general planar network can be written by inspection
without going through the detailed steps.

The general mesh equations for three mesh resistive network can be written as

𝑅11 𝐼1 − 𝑅12 𝐼2 − 𝑅13 𝐼3 = 𝑉𝑎

−𝑅21 𝐼1 + 𝑅22 𝐼2 − 𝑅23 𝐼3 = 𝑉𝑏

−𝑅31 𝐼1 − 𝑅32 𝐼2 + 𝑅33 𝐼3 = 𝑉𝑐

By comparing the above sets of equations, we observe that, we have to take into
account

1. The self resistance in each mesh


2. The mutual resistance between all pairs of meshes and
2. The algebraic sum of the voltages in each mesh.

The self resistance of loop 1, 𝑅11 = 𝑅1 + 𝑅2 + 𝑅5 is the sum of the resistances


through which 𝐼1 passes. It is positive. The mutual resistance of loops 1, 2 𝑅12 =
−𝑅5 is the sum of the resistances common to loop currents 𝐼1 and 𝐼2 and the mutual
resistance of loops 1, 3 𝑅13 = −𝑅2 is the sum of the resistances common to loop
currents 𝐼1 and 𝐼3 . If the directions of the currents passing through the common
resistance are the same, the mutual resistance will have a positive sign: and if the
directions of the currents passing through the common resistance are opposite, then
the mutual resistance will have a negative sign. 𝑉𝑎 = 𝑉1 − 𝑉2 is the voltage which
drives loop 1. Here, the positive sign is used if the direction of the current is the
same as the direction of the source. If the direction of the current is opposite to the
direction of the source, then the negative sign is used.
Similarly 𝑅22 = 𝑅3 + 𝑅5 + 𝑅6 and 𝑅33 = 𝑅2 + 𝑅3 + 𝑅4 are the self resistances
of loops two and three respectively. The mutual resistances 𝑅21 = − 𝑅5 , 𝑅23 = −𝑅3 ,
𝑅31 = −𝑅2 , 𝑅32 = −𝑅3 are the sums of the resistances common to the mesh

3
currents indicated in their subscripts. 𝑉𝑏 = 𝑉2 , 𝑉𝑐 = 0 are the sum of the voltages
driving their respective loops.

Planar and non-planar circuits

A planar circuit is one which can be drawn on a plane surface without crossovers. A
non-planar circuit is one which cannot be drawn on a plane surface without
crossovers. Figures 2.2(a) and (c) show a planar circuit while Figure 2.2(b) shows a
non-planar network.

+ -

+
-
+
+ + + -
- - -

(c)
(a) (b)

Figure 2.2 (a) and (c) planar networks (b) A non-planar network

Example 2.1 Two batteries are connected in parallel. The emf and internal
resistance of one are 110V and 6 Ω respectively and the corresponding values for
other are 130V and 4 Ω respectively. A resistance of 20 Ω is connected across the
parallel combination. Calculate i) the value and direction of the current in each
battery ii) the terminal voltage.

Solution: For the given description, the circuit is shown in Figure 2.3

6 4
I1
110V
I2 20
130V

Figure 2.3 Circuit for Example 2.1

Applying KVL to the first mesh, we have

−110 + 6𝐼1 + 4(𝐼1 − 𝐼2 ) + 130 = 0 or 5𝐼1 − 2𝐼2 = −10

Applying KVL to the second mesh, we have

4(𝐼2 − 𝐼1 ) + 20𝐼2 − 130 = 0 or − 2𝐼1 + 12𝐼2 = 65

Solving the above equations, we get 𝐼1 = 0.1785A and 𝐼2 = 5.446A

4
The directions of the currents are outward from positive terminal as marked in the
circuit. The terminal voltage is equal to

110 − 6 × 0.1785 or 130 − 4 × (5.446 − 0.1785) or 20 × 5.446 = 108.92V

Example 2.2 Find the currents in the resistances in the circuit shown in Figure 2.4(a)
using mesh analysis.

10Ω 30Ω 10Ω 30Ω

20V 20Ω 30V 20V I1 20Ω I2 30V

(a) (b)

Figure 2.4 (a) Circuit for Example 2. 2 (b) Circuit with loop currents marked

Solution: The KVL equation for loop 1 in Figure 2.4(b) is

10𝐼1 + 20(𝐼1 − 𝐼2 ) − 20 = 0, i. e. , 30𝐼1 − 20𝐼2 = 20

The KVL equation for loop 2 in Figure 2.4(b) is

30𝐼2 + 30 + 20(𝐼2 − 𝐼1 ) = 0, i. e., −20𝐼1 + 50𝐼2 = −30

The above two mesh equations in matrix form are

30 −20 𝐼1 20
[ ][ ] = [ ]
−20 50 𝐼2 −30

Using Cramer’s rule

20 −20 30 −20
𝐼1 = | |÷| | = 0.364A
−30 50 −20 50
30 20 30 −20
And 𝐼2 = | |÷| | = −0.454A
−20 −30 −20 50
So the loop currents 𝐼1 and 𝐼2 are 𝐼1 = 0.364A ; 𝐼2 = −0.454A

So the currents in the 10Ω, 30Ω and 20Ω resistances respectively are

𝐼1 = 0.364A, 𝐼2 = −0.454A, and 𝐼1 − 𝐼2 = 0.364 − (−0.454) = 0.818A.

Example 2.3 Determine the current supplied by each battery in the circuit shown in
Figure 2.5(a).

5
4Ω 6Ω 4V 10Ω 6Ω 4V 10Ω

2Ω 8Ω 2Ω 8Ω
10V 6V 10V i1 i2 i3 6V
8V 2V 8V 2 2V
1 3

12Ω 12Ω
(a) (b)

Figure 2.5 (a) Circuit for Example 2. 3 (b) Circuit with loop currents marked

Solution: Let the loop currents in the three meshes be 𝑖1 , 𝑖2 and 𝑖3 respectively as
shown in Figure 2.5(b).

The KVL equation around loop 1 is

4𝑖1 + 2(𝑖1 − 𝑖2 ) + 8 − 10 = 0, i. e. , 6𝑖1 − 2𝑖2 = 2

The KVL equation around loop 2 is

6𝑖2 − 4 + 8(𝑖2 − 𝑖3 ) − 2 + 12𝑖2 − 8 + 2(𝑖2 − 𝑖1 ) = 0, i. e. , −2𝑖1 + 28𝑖2 − 8𝑖3 = 14

The KVL equation around loop 3 is

10𝑖3 + 6 + 2 + 8(𝑖3 − 𝑖2 ) = 0, i. e., −8𝑖2 + 18𝑖3 = −8

The above three loop equations in matrix form are

6 −2 0 𝑖1 2 3 −1 0 𝑖1 1
[−2 28 −8] [𝑖2 ] = [ 14 ] , i. e., [−1 14 −4] [𝑖2 ] = [ 7 ]
0 −8 18 𝑖3 −8 0 −4 9 𝑖3 −4

Using Cramer’s rule

1 −1 0 3 −1 0 157
∴ 𝑖1 = | 7 14 −4| ÷ |−1 14 −4| = = 0.489A
321
−4 −4 9 0 −4 9
3 1 0 3 −1 0 150
𝑖2 = |−1 7 −4| ÷ |−1 14 −4| = = 0.467A
321
0 −4 9 0 −4 9
3 −1 1 3 −1 0
76
𝑖3 = |−1 14 7 | ÷ |−1 14 −4| = − 321 = −0.236A
0 −4 −4 0 −4 9
Referring to Figure 2.5(b)

Discharge current of 10V source = 𝑖1 = 0.489A

Charging current of 8V source = 𝑖1 − 𝑖2 = 0.489 − 0.467 = 0.022A

Discharge current of 6V source = −𝑖3 = 0.236A

6
Discharge current of 4V source = 𝑖2 = 0.467A

Discharge current of 2V source = 𝑖2 − 𝑖3 = 0.467— (−0.236) = 0.703A

2.3 NODAL ANALYSIS

In nodal analysis, KCL is used to write the equilibrium equations. In general, in an N


node network, one of the nodes is selected as a reference or datum node and 𝑁 − 1
nodal equations are written by assuming 𝑁 − 1 node voltages and those equations
are solved to obtain node voltages. From the knowledge of node voltages, required
branch currents may be computed. Each node in a circuit can be assigned a number
or a letter. The node voltage is the voltage of a given node with respect to the
reference node, which we assume at zero potential. We can arbitrarily choose any
node as the reference node. However it is convenient to choose the node with most
connected branches as the reference node.

Steps to be followed in nodal analysis

1. Assuming that a network has n nodes, take one node as a reference node.
Always assume that any node under consideration is always at a higher potential
compared to other nodes so that all the currents are leaving that node.

2. Apply KCL at each node except for the reference node and apply ohm’s law to
the branch currents.

2. Solve the simultaneous equations for the unknown node voltages.

4. Using these voltages find any branch current required.

In the circuit shown in Figure 2.6, node 3 is assumed as the reference node. The
voltages at nodes 1 and 2 are with respect to node 3.

R2

I4 I5
R3
1 V1 V2 2

I8
I2 I3 I6
R4 I7
I1 R1 R5

Vs

Figure 2.6 A general circuit, KCL at node 1, KCL at node2

Applying Kirchhoff’s current law(KCL) at node1 in Figure 2.6, i.e. the current
entering the node is equal to the current leaving the node, we have

7
𝑉1 𝑉1 − 𝑉2 𝑉1 − 𝑉2
𝐼1 = 𝐼2 + 𝐼3 + 𝐼4 , 𝑖. 𝑒. 𝐼1 = + +
𝑅1 𝑅2 𝑅3

where 𝑉1 and 𝑉2 are the voltages at nodes 1 and 2 respectively.

Similarly KCL at node2 in Figure 2.6 gives

𝑉2 − 𝑉1 𝑉2 − 𝑉1 𝑉2 − 𝑉𝑠 𝑉2
𝐼5 + 𝐼6 + 𝐼7 + 𝐼8 = 0, i. e. + + + =0
𝑅2 𝑅3 𝑅4 𝑅5

Rearranging the above equations, we have

1 1 1 1 1
𝑉1 ( + + ) − 𝑉2 ( + ) = 𝐼1
𝑅1 𝑅2 𝑅3 𝑅2 𝑅3

1 1 1 1 1 1 𝑉𝑆
and − 𝑉1 ( + ) + 𝑉2 ( + + + )=
𝑅2 𝑅3 𝑅2 𝑅3 𝑅4 𝑅5 𝑅4

From the above equations, we can find the voltage at each node.

2.4 NODAL EQUATIONS BY INSPECTION METHOD

The nodal equations for a general planar network can also be written by inspection
method without going through the detailed steps. Consider the 3 node network
shown in Figure 2.7. Node c is taken as the reference node.

R3
R1 R5
a Va Vb b

R2
R4
V1 I1 V3

V2

Figure 2.7 A general circuit

The KCL at node a gives

𝑉𝑎 − 𝑉1 𝑉𝑎 − 𝑉𝑏 𝑉𝑎 − 𝑉𝑏
+ + − 𝐼1 = 0,
𝑅1 𝑅3 𝑅2

The KCL at node b gives

𝑉𝑏 − 𝑉𝑎 𝑉𝑏 − 𝑉𝑎 𝑉𝑏 − 𝑉2 𝑉𝑏 − 𝑉3
+ + + =0
𝑅2 𝑅3 𝑅4 𝑅5

8
Rearranging the above equations, we get

1 1 1 1 1 𝑉1
𝑉𝑎 ( + + ) − 𝑉𝑏 ( + ) = 𝐼1 +
𝑅1 𝑅2 𝑅3 𝑅2 𝑅3 𝑅1

1 1 1 1 1 1 𝑉2 𝑉3
−𝑉𝑎 ( + ) + 𝑉𝑏 ( + + + )= +
𝑅2 𝑅3 𝑅2 𝑅3 𝑅4 𝑅5 𝑅4 𝑅5

In general, the above equations can be written as

𝐺𝑎𝑎 𝑉𝑎 + 𝐺𝑎𝑏 𝑉𝑏 = 𝐼1

And 𝐺𝑏𝑎 𝑉𝑎 + 𝐺𝑏𝑏 𝑉𝑏 = 𝐼2

Comparing the above two sets of equations, the self conductance at node a,
1 1 1
𝐺𝑎𝑎 = (𝑅 + 𝑅 + 𝑅 ) is the sum of the conductances connected to node a.
1 2 3

1 1 1 1
Similarly, 𝐺𝑏𝑏 = (𝑅 + 𝑅 + 𝑅 + 𝑅 ) is the sum of the conductances connected to node
2 3 4 5
1 1
b. 𝐺𝑎𝑏 = − (𝑅 + 𝑅 ) is the sum of the mutual conductances connected to node a
2 3
1 1
and node b. Similarly 𝐺𝑏𝑎 = − (𝑅 + 𝑅 ) is the sum of the mutual conductances
2 3
connected to node b and node a. All the self conductances have positive sign and all
the mutual conductances have negative sign. 𝐼1 and 𝐼2 are the sum of the source
currents at nodes a and b respectively. The current which drives into the node has
positive sign, while the current which drives away from the node has negative sign.

2.5 NODAL VERSUS MESH ANALYSIS

The analysis of a complex circuit can usually be accomplished by either the node
voltage or mesh current method. The choice of either mesh or nodal analysis to a
particular problem depends on the number of voltage sources or current sources
present in the network.

When a circuit contains only voltage sources or a large number of voltage


sources, it is probably easier to use the mesh current method. Conversely when the
circuit contains only current sources or more current sources, it is preferable to use
nodal analysis. Also a network with fewer nodes than meshes is better analyzed
using nodal analysis, while a circuit with fewer meshes than nodes is better analyzed
using mesh analysis. In other words, the best technique is one which gives smaller
number of equilibrium equations.

The choice between nodal and mesh analysis may also be based on the
information required. If node voltages are required, it may be advantageous to apply
nodal analysis. On the other hand, if several currents are to be known, it may be
wise to proceed with mesh current analysis.

9
Mesh analysis is applicable only for planar networks whereas nodal
analysis can be applied to planar as well as non-planar networks. A network is said
to be planar, if it can be drawn on a plane surface without crossovers. A non-planar
network cannot be drawn on a plane surface without a crossover.

It is often advantageous to know both the methods. The first advantage lies
in the fact that the results obtained by the first method can be verified by the second
method. Also, both the methods have limitations. For example while analyzing a
transistor circuit, only mesh method is suited and while analyzing an op-amp circuit,
only nodal method is suited.

Example 2.4 Two batteries having emf of 20V and 8V and internal resistances of 4Ω
and 2Ω respectively are connected in parallel across a load resistance of 6Ω.
Calculate (a) the individual battery currents, (b) the current through the load, (c) the
voltage across the load, and (d) the power consumed in the load and the power
supplied by the sources. Also show that the power supplied by the sources is equal
to the power dissipated by resistors.

I1 1 I2

I3
20V 8V

4Ω 2Ω

Figure 2.8 Circuit for Example 2.4

Solution: In the given circuit shown in Figure 2.8, let the voltage at the single node
be 𝑉1. Using nodal analysis at node1, we have

𝑉1 − 20 𝑉1 𝑉1 − 8
𝐼1 + 𝐼2 + 𝐼3 = 0, i. e. , + + = 0 or 𝑉1 = 9.818V
4 6 2
(a) The battery currents are

𝑉1 − 20 9.818 − 20 𝑉1 − 8 9.818 − 8
𝐼1 = = = −2.545A and 𝐼2 = = = 0.909A
4 4 2 2
(b) The current through the load is
𝑉1 9.818
𝐼3 = = = 1.636A , 𝐼1 + 𝐼2 + 𝐼3 = 0
6 6

or 𝐼3 = −(𝐼1 + 𝐼2 ) = −(−2.545 + 0.909) = 1.636A

(c) The voltage across the load is = 𝐼3 × 6 = 1.636 × 6 = 9.816V

10
(d) The power consumed in the load = 𝐼32 × 6 = (1.636)2 × 6 = 16.06W

The power supplied by the 20V battery = 20 × (−𝐼1 ) = 20 × 2.545 = 50.9W

The power absorbed by the 8V battery = 8𝐼2 = 8 × 0.909 = 7.27W

(e) The power consumed in 4Ω resistor = 𝐼12 × 4 = (−2.545)2 × 4 = 25.908W

The power consumed in 2Ω resistor = 𝐼22 × 2 = (0.909)2 × 2 = 1.652W

Power supplied by sources = 50.9 − 7.27 = 42.63W

Power consumed in resistors = 16.06 + 25.908 + 1.652 = 42.63W

Hence power supplied by sources = power consumed by resistors

Example 2.5 Find 𝑉1 and 𝑉2 in the circuit shown in Figure 2.9

4V
1 6? 2

12A 4? 4A 8?

0
Figure 2.9 Circuit for Example 2.5

Solution: The given network shown in Figure 2.9 has two nodes in addition to the
ground node. Let the voltages at nodes 1 and 2 be 𝑉1 and 𝑉2 respectively.

Applying KCL at node 1, we get

𝑉1 𝑉1 − 4 − 𝑉2 1 1 1 4
+ = 12 i. e. , 𝑉1 ( + ) − 𝑉2 ( ) = 12 +
4 6 4 6 6 6
or 5𝑉1 − 2𝑉2 = 152

Applying KCL at node 2, we get

𝑉2 𝑉2 + 4 − 𝑉1 1 1 1 4
+ = 4 i. e. , −𝑉1 ( ) + 𝑉1 ( + ) = 4 −
8 6 6 8 6 6
or − 4𝑉1 + 7𝑉2 = 80

Solving the above equations, we have

𝑉1 = 45.93V and 𝑉2 = 37.33V

Example 2.6 Using nodal analysis, find 𝐼 in the circuit shown in Figure 2.10

11
4A

1 2
I 4Ω

2A 6Ω 6A
4V

Figure 2.10 Circuit for Example 2. 6

Solution: Let the voltages at nodes 1 and 2 of the circuit in Figure 2.10 be 𝑉1 and 𝑉2
respectively.

Using nodal analysis at node 1, we get

𝑉1 𝑉1 − 𝑉2 1 1 𝑉2
+ =2+4 , i, e. , 𝑉1 ( + ) − = 6
6 4 6 4 4
or 5𝑉1 − 3𝑉2 = 72

Using nodal analysis at node 2, we get

𝑉2 − 𝑉1 𝑉2 − 4 𝑉1 𝑉2 𝑉2 4 5
+4+ =6 , i. e. , − + + =6−4+ =
4 8 4 4 8 8 2
or − 2𝑉1 + 3𝑉2 = 20

Solving the above equations, we get

𝑉1 = 30.666V and 𝑉2 = 27.107V

𝑉1 − 𝑉2 30.667 − 27.107
∴ 𝐼= = = 0.89A
4 4
Example 2.7 Find the nodal voltages and the power delivered by the 2A current
source in the circuit shown in Figure 2.11, using nodal analysis. Verify the result
using mesh analysis.

1A
20 V2 10
V1 V3

2A

5 15

Figure 2.11 Circuit for Example 2.7

12
Solution: Writing nodal equations for the given circuit of Figure 2.11, we have

The KCL equation at node 1 is

𝑉1 𝑉1 − 𝑉2
+ +1=0 or 5𝑉1 − 𝑉2 = −20
5 20
The KCL equation at node 2 is

𝑉2 − 𝑉1 𝑉2 − 𝑉3
+ −2=0 or − 𝑉1 + 3𝑉2 − 2𝑉3 = 40
20 10
The KCL equation at node 3 is

𝑉3 𝑉3 − 𝑉2
+ −1=0 or − 3𝑉2 + 5𝑉3 = 30
15 10
Solving the above three node equations, we have the node voltages

𝑉1 = 2V ; 𝑉2 = 30V ; 𝑉3 = 24V

The power delivered by the 2A source = 𝑉2 × 2 = 30 × 2 = 60W

SHORT QUESTIONS AND ANSWERS

1. Basically how many types of energy sources are there? Name them.
A. Basically there are two types of energy sources:
1. Voltage sources 2. Current Sources

2. What does the practical voltage source consist of?


A. The practical voltage source consists of an ideal voltage source in series with
an internal resistance.

3. What does the practical current source consist of?


A. The practical current source consists of an ideal current source in parallel with
an internal resistance.

4. How do you convert a voltage source into a current source?


A. A voltage source of 𝑉𝑠 in series with an internal resistance 𝑅𝑠 can be
converted into a current source of 𝐼𝑠 = 𝑉𝑠 ⁄𝑅𝑠 in parallel with an internal
resistance 𝑅𝑠 .

5. How do you convert a current source into a voltage source?


A. A current source of 𝐼𝑠 in parallel with an internal resistance 𝑅𝑠 can be
converted into a voltage source of 𝑉𝑠 = 𝐼𝑠 𝑅𝑠 in series with an internal
resistance 𝑅𝑠 .
6. Source conversion is based on what?
A. Source conversion is based on the concept of equivalence.

13
7. What is an equivalent circuit?
A. An equivalent circuit is one whose terminal characteristics remain identical to
the original circuit.

8. Two voltage sources 𝑉1 and 𝑉2 connected in series are equivalent to what?


A. A voltage source of 𝑉1 + 𝑉2 .

9. Two current sources 𝐼1 and 𝐼2 connected in parallel are equivalent to what?


A. A current source of 𝐼1 + 𝐼2 .

10. Can two voltage sources 𝑉1 and 𝑉2 be connected in parallel?


A. Two voltage sources 𝑉1 and 𝑉2 can be connected in parallel only when 𝑉1 =
𝑉2 .

11. Can two current sources 𝐼1 and 𝐼2 be connected in series?


A. Two current sources 𝐼1 and 𝐼2 can be connected in series only when 𝐼1 = 𝐼2.

12. Why any element connected in parallel with a voltage source be neglected?
A. Ideally a voltage source has to supply constant output voltage irrespective of
the current drawn from it. So any thing connected in parallel with a voltage
source becomes trivial and can be neglected.

12. Why any element connected in series with a current source be neglected?
A. Ideally a current source has to supply constant output current irrespective of
the load. So anything connected in series with a current source becomes
trivial and can be neglected.

13. What is a loop?


A. A loop is a closed path drawn starting at a node and tracing the path such that
we return to the original node without passing an intermediate node more than
once.

14. What is a mesh?


A. A mesh is a special case of a loop. A mesh is defined as a loop which does
not contain any other loops within it.

15. If a circuit has 𝐵 branches and 𝑁 nodes including reference node, how many
meshes are there?
A. Number of meshes = Branches – (Nodes − 1), i. e. , 𝑀 = 𝐵 − (𝑁 − 1).

16. What is a planar circuit?


A. A planar circuit is one which can be drawn on a plane surface without
crossovers.

17. What is a non-planar circuit?

14
A. A non-planar circuit is one which cannot be drawn on a plane surface without
cross overs.

18. In mesh analysis, which law is used to write the equilibrium equations?
A. In mesh analysis, KVL is used to write the equilibrium equations.

19. In nodal analysis, which law is used to write the equilibrium equations?
A. In nodal analysis, KCL is used to write the equilibrium equations.

20. What is node voltage?


A. The node voltage is the voltage of a given node, with respect to the reference
node, which is assumed at zero potential.

21. When is mesh analysis preferred over nodal analysis?


A. Mesh analysis is preferred over nodal analysis
1. When the circuit contains only voltage sources or a large number of
voltage sources.
2. When the circuit contains fewer meshes than nodes.
3. When several currents are to be determined.
However for mesh analysis, network must be planar.

22. When is nodal analysis preferred over mesh analysis?


A. Nodal analysis is preferred over mesh analysis
1. When the circuit contains only current sources or more current sources.
2. When the circuit contains fewer nodes than meshes.
3. When node voltages are required.
4. When the network is non-planar only nodal analysis can be used.

23. If two voltage sources 𝑉1 and 𝑉2 are connected in parallel and series
respectively, the correct relation is---

24. If two current sources 𝐼1 and 𝐼2 are connected in series and parallel
respectively, the correct relation is---

25. Two graphs depicted here represent ideal sources. Identify the two graphs.

i i

v v

(i) (ii)

26. The powers delivered by the battery and the current source are respectively

15
8V 5A

27. If 𝐼1 = 8A, 𝑉𝑋𝑌 is equal to

20V 10I1

X - + + - Y

28. The equivalent circuit of the Figure shown below is


8V
6A

29. The current source transformation of branch AB is given by


A

8Ω 16V

30. The equivalent parallel combination of the circuit shown is given by

16V
8V



2A

31. The voltage across terminals A − B is

16
10V

A

16V 8A 4Ω

32. If a network contains B branches, and N nodes, then the number of mesh
current equations would be

33. A network has 8 nodes and 4 independent loops. The number of branches in
the network is

PROBLEMS

2.1P Using mesh analysis, find the current delivered by the source in the circuit
shown in Figure 2.1P. Verify the result using nodal technique.

30Ω

10Ω 60Ω

+
100V 20Ω 15Ω
-

Figure 2.1P

𝟐. 𝟐𝐏 Determine the value and direction of the current in the 20Ω resistor shown in
Figure 2.2P

Ω
10
50
Ω

20Ω
30

Ω
Ω

40

10V

Figure 2.2P

17
2.3P Determine the current in the 6Ω resistor in the circuit shown in Figure 2.3P

4Ω 4Ω


20V 6V
8V

2Ω 2Ω

10V 8Ω

Figure 2.3P

2.4P Find the voltage 𝑣 using nodal technique for the circuit shown in Figure 2.4P

+ -

8V
4Ω b 6Ω
a c
+ v -

+
8Ω 24V 4A
-

Figure 2.4P

2.5P Refer the network shown in Figure 2.5P. Find the currents 𝐼1 and 𝐼2 using
nodal analysis.

2O

2A i1 2O i2 2O 2A

Figure 2.5P

18
NETWORK THEOREMS

2.6 INTRODUCTION
We have discussed the loop and node analysis of electric networks earlier.
Any network can be analyzed by either of these methods. As a further aid to network
analysis, many network theorems have been developed. Network theorems are very
useful for simplifying complicated electric circuits. Each of these theorems has some
specific applications. The use of these theorems for these applications can yield
quick results. Moreover, these theorems provide an insight into the behaviour of
electric networks.
These theorems are valid for dc networks as well as ac networks. In dc
networks we have to deal with resistances only(In dc networks inductances and
capacitances are important only under transient conditions. In steady state, the
currents and voltages depend on resistance only). All voltages, currents, resistances,
and conductances in dc circuits are real quantities only. However, in ac networks we
have to deal with impedances. All voltages, currents, impedances and admittances
in ac networks are complex quantities. Therefore, in ac networks, all mathematical
operations must be carried out by following the rules of phasor algebra. Here we
discuss the application of superposition theorem, Thevenin’s theorem, Norton’s
theorem and maximum power transfer theorem to dc networks only. These
theorems are applicable to a large category of circuits and are simple in application
and hence help in reducing both time and labour in the analysis of circuits.

2.7 SUPERPOSITION THEOREM

Superposition theorem states that in a linear bilateral network containing N


independent sources, each branch current (branch voltage) is the sum of N currents
(voltages), each of which is determined by considering one independent source at a
time and removing all other sources. In removing the sources, independent voltage
sources are short circuited (or replaced by the series internal resistances of non-
ideal voltage sources), while the independent current sources are open circuited (or
replaced by the parallel internal resistances in the case of non-ideal current
sources).
Superposition theorem is valid to compute only the voltage and current
responses. It is not valid for power responses.
Procedure to find the current through a resistance using superposition theorem
1. Find the current through the resistance when only one independent source is
acting, replacing all other independent sources by respective internal
resistances.
2. Find the current through the resistance for each of the independent sources.
3. Find the resultant current through the resistance by finding the algebraic sum of
all the currents.
Illustration with an example
This theorem can be better understood with a numerical example.

19
Consider the circuit of Figure 2.12(a), which contains two sources. Suppose we want
to find the current through the 12Ω resistance in the circuit by superposition theorem.
This can be done as follows:

8 6 8 6

8 12  16V 8 12 
I1

Figure 2.12 (a) Circuit for example (b) Circuit with only 8A source.

Step 1 First consider only the 8A source and deactivate the 16V source as shown
in Figure 2.12 (b) and find I1 , the current in 12Ω due to only 8A source.

6
I1 = 8  = 2.667A
6 + 12
Step 2 Next consider only the 16V source and deactivate the 8A source as shown
in Figure 2.13(a) and find I 2 , the current in 12Ω due to only 16V source.

16
I2 =
= 0.889A
12 + 6
Step 3 Find the algebraic sum of the current I1 and I 2

I = I1 + I 2 = 2.667 + 0.889 = 3.556A


This is the total current passing through the 12Ω resistor when both the sources are
acting simultaneously.

Let us verify the above result by loop analysis.

8 6 8 6

I2 8 12  16V
12  16V I1 I2

(b)
(a)

Figure 2.13 (a) Given Circuit with only 16V source (b) Circuit for loop analysis

In Figure 2.13(b) I1 = 8A

The only loop equation is 6 I 2 + 16 + 12 ( I 2 − 8) = 0 or 18 I 2 = 80

20
80
 I2 == 4.444A
18
 Current through the 12Ω resistor by loop analysis is

I = I1 − I 2 = 8 − 4.444 = 3.556A
So the superposition theorem is verified.

Let us now examine it for power responses.

Power dissipated in the 12Ω resistance due to current source acting alone.

P1 = I12 12 = 2.6672 12 = 85.35W


Power dissipated in the 12Ω resistance due to voltage source acting alone.

P2 = I 22 12Ω
= 0.8892 12 = 9.484W
Power dissipated in the 12 resistance when both the sources are acting
simultaneously is

P = I 2 12 = 3.5562 12 = 151.74W


From the above results, the superposition of the individual powers P1 and P2 gives

P1 + P2 = 85.35 + 9.484 = 94.834W


which is not equal to P=151.74W

This shows that the superposition theorem is not valid for power responses. It is valid
only for computing current and voltage responses.

Example 2.8 Applying the principle of superposition calculate the current flowing in
the 6Ω resistance in the circuit shown in Figure 2.14. Show that the superposition
principle is valid only for voltage and current responses and is not valid for power
response.
6 V 2

8V
+ 4A
- 4

Figure 2.14 Circuit for Example 2.8


Solution: Step I First consider only the 8V voltage source and remove the 4A
current source (open circuit it) as shown in Figure 2.15(a) and find the current I1 in
the 6Ω resistor.
Since all the three resistors are in series, the current through the 6Ω resistor, I1 is
8
I1 = = 0.6667A
6+2+4

21
6 2 6 2

I1 I2

8V
+ OC SC 4A
- 4 4

(a) (b)

Figure 2.15 Circuit with (a) Current source open circuited (b) Voltage source short
circuited
Step II Then considering only the 4A current source and neglecting the 8V voltage
source as shown in Figure 2.15(b), find I 2
2+4
I2 = 4  = 2A
2+4+6
Step III Find the algebraic sum of the currents I1 and I 2
Since I1 and I 2 are in opposite directions in the 6Ω resistor, the current through the
6Ω resistor is I 2 − I1 = 2 − 0.6667 = 1.3333A from right to left.
Let us verify the above result by applying nodal analysis.
The current passing through the 6Ω resistor must be 1.3333A .
Applying nodal analysis to Figure 2.14, we have
V −8 V
+ =4
6 2+4
1 1 8
V  +  = 4+
6 6 6
 V = 16V
Therefore, current in the 6Ω resistor due to both sources acting simultaneously is
V − 8 16 − 8
I= = = 1.3333A
6 6
So the superposition theorem is verified. Let us now examine the power responses.
Step IV Verification for power responses
Power dissipated in the 6Ω resistor due to 8V voltage source acting alone.
P1 = I12 R = ( 2 ) 6 = 24W
2

Power dissipated in the 6Ω resistor due to 4A current source acting alone


P2 = I 22 R = ( 0.667 ) 6 = 2.669W
2

Power dissipated in the 6Ω resistor when both the sources are acting
simultaneously is given by
P = I 2 R = (1.333) 6 = 10.661W
2

From the above results, the superposition of P2 and P1 gives


𝑃1 + 𝑃2 = 24 + 2.669 = 26.669W
which is not equal to P = 10.661W

22
This result shows that the superposition theorem is not valid for power response. It is
valid only for voltage and current responses.

Example 2.9 Find the voltage across the 5Ω resistor in Figure 2.16 by using the
superposition theorem.
10 A 5 B

8 12
+
15V 15
- + 5A
30V
-
Figure 2.16 Circuit for Example 2.9

Solution: Step I First consider only the 15V source, deactivating the other two
sources as shown in Figure 2.17(a) and find the voltage across the 5Ω resistor V1.
10 5 10 5
A A
+ - - +
8
+ 8
15V 15 15
+
- 30V
-

(a) (b)

10 5
- +
12
15 8

5A

(c)
Figure 2.17 (a) Circuit with only 15V source (b) Circuit with only 30V source
(c) Circuit with only 5A source
Assuming a voltage V at node A as shown in Figure 2.17(a), the KCL equation is
V − 15 V V
+ + =0
10 15 5 + 8
 1 1 1  15
i.e. V + + =
 10 15 13  10
or V = 6.15V
The voltage across the 5Ω resistor due to the 15V source is
V 5
V1 =  5 = 6.15  = 2.365V
5+8 5+8

23
Step II Our second step is to find the voltage V2 across the 5Ω resistor due to the
30V source only, while other sources are set equal to zero. The circuit is redrawn as
shown in Figure 2.17(b).
Assuming a voltage V at node A, as shown in Figure 2.17(b), the KVL equation is
V V V − 30
+ + =0
10 15 5 + 8
 1 1 1  30
i.e., V  + +  =
 10 15 13  13
or V = 9.473V
The voltage across the 5Ω resistor due to the 30V source is
 V − 30   9.473 − 30 
V2 =  5 =    5 = −7.895V
 5+8   5+8 
Step III
The next step is to find the voltage V3 across the 5Ω resistor due to the 5A current
source only, while the other sources are set equal to zero. The circuit is redrawn as
shown in Figure 2.17(c).
8
Current in 5Ω resistor = 5 × 8+5+(10 =2.105A
⃦15)
The voltage across the 5Ω resistor is V3 = 2.105  5 = 10.525V
Step IV Find the algebraic sum of the responses V1 ,V2 ,V3 across the 5Ω resistor.
V = V1 + V2 + V3 = 2.365 − 7.895 − 10.525 = −16.055V
The negative sign of the voltage indicates that the voltage at A is at lower potential
w.r.t. B.

Example 2.10 Determine the current through the 6 Ω resistor in the circuit shown in
Figure 2.18 using superposition theorem.
4 6

8A 2
4 12V

Figure 2.18 Circuit for Example 2.10

Solution: Step I First consider only the 12V source and deactivate the 8A current
source, i.e. open circuit it and find the current I1 in the 6 Ω resistor. The circuit then
becomes as shown in Figure 2.19(a) and its equivalent is shown in Figure 2.19(b).
12
I1 = = 1.5A
6+2

24
4 6
6
I1
I1
2 4 2 12V
12V

(b)
(a)

Figure 2.19 Circuit with 8A source deactivated


Step II Next considering only the 8A source and deactivating the 12V source, i.e.
short circuiting it, as shown in Figure 2.20(a), find the current I 2 in the 6 Ω resistor.
2
I2 = 8  = 2A
2+6
4 6 4 6

I2
2 8A 2
8A 4

(a) (b)

Figure 2.20 Circuit with 12V source deactivated


Step III Find the algebraic sum of the responses I1 and I 2
Since I1 and I 2 are in opposite directions, we get the resultant current through the 6 Ω
resistor
I = I 2 − I1 = 2 − 1.5 = 0.5A
from right to left.

Example 2.11 Find the current ix in the circuit shown in Figure 2.21(a) using
superposition theorem.
i x I 1

36Ω 12Ω 36Ω 12Ω

+ 8Ω
+ 8Ω
90V 2A 90V
+
60V
- -
-

(a) (b)

25
I 2 I 3

12Ω
36Ω
8Ω 12Ω
+ 36Ω 2A 8Ω
60V
-

(c) (d)
Figure 2.21 (a) Circuit for Example 2.
(b), (c), (d) Circuit with only one source activated at a time.

Solution: Step I First consider only the 90V voltage source and deactivate the other
two sources of 2A and 60V as shown in Figure 2.21(b) and calculate the current I1 .
90 12
𝐼1 = { }× = 1.32𝐴
36 + (12 ⃦8) 12 + 8
Step II Next consider only the 60V voltage source and deactivate the other two
sources of 2A and 90V as shown in Figure 2.21(c) and calculate the current I 2 .
60 36
𝐼2 = ( )× = 2.647𝐴
12 + (36 ⃦8) 36 + 8
Step III Next consider only the 2A current source and deactivate the other two
voltage sources of 90V and 60V as shown in Figure 2.21(d) and calculate the current
I3 .
(36 ⃦12)
𝐼3 =2× (36 = 1.058𝐴
⃦12)+8
Step IV Find the algebraic sum of the responses I1 , I 2 and I 3 .
 ix = I1 + I 2 + I 3 = 1.32 + 2.647 + 1.058 = 5.029A
2.8 THEVENIN’S THEOREM
In some cases, only one particular resistance in a circuit is varied and it is required to
determine only the value of current through it or voltage across it for various values
of that element. So there is no need to analyze the complete circuit. Thevenin’s
theorem is extensively used in such cases. The use of this theorem enables us to
replace the original circuit by a simple equivalent circuit.
Thevenin’s theorem states that any linear, active, bilateral two terminal
network containing a number of resistances and voltage sources and/or current
sources can be replaced by a simple equivalent circuit consisting of a single voltage
source VTh in series with a single resistance RTh .The Thevenin equivalent voltage VTh
is the open circuit voltage across the two terminals of the network and the Thevenin
resistance RTh is the resistance measured between the two terminals when all the
energy sources are replaced by their internal resistances.
Figure 2.22(a) shows a linear network that contains resistances, voltage sources
and/or current sources with output terminals AB. Using Thevenin’s theorem, the

26
linear network can be replaced by a single voltage source VTh in series with a single
resistance RTh as shown in Figure 2.22(b). Now any resistance RL can be connected
between the terminals AB and the current through it can be obtained easily.
VTh
I= . VThis also denoted by VOC .
RTh + RL
R Th
A
A
Linear
+
Network V
B
-
Th

B
(a) (b)
Figure 2.22 (a) Any linear network (b) Its Thevenin equivalent.

Procedure to find the current through a branch using Thevenin’s theorem

1. Remove, i.e., open the branch through which current is to be found and mark the
terminals AB.
2. Calculate the open circuit voltage VTh between the terminals AB.
3. Replace the independent sources with their internal resistances.
(If the internal resistances are zero, then the voltage source is to be replaced by
short circuit and current source is to be replaced by open circuit).
3. Calculate RTh between the terminals AB.
5. Replace the network by Thevenin voltage source in series with Thevenin
resistance with output terminals AB.
6. Connect the removed branch between the terminals AB and find current through
it.
VTh
IL =
RTh + RL
If a circuit contains no independent energy sources, VTh is zero and the
circuit has only the equivalent Thevenin resistance.
Illustration of Thevenin’s theorem with an example.

Consider a simple network shown in Figure 2.23


6 4
A

18V 12  R
L
8

B
(a)

Figure 2.23 Circuit for illustration of Thevenin’s theorem

27
To find the current through the load resistor 8Ω connected between terminals A-
B using Thevenin’s theorem, follow the following steps.

6 4 6 4 R Th
A A A
8
R L

R 8
18V 12  V oc 12  Th
V Th
12V

B B B
(a) (b)

Figure 2.24(a) Circuit to find 𝑉𝑇ℎ (b) Circuit to find 𝑅𝑇ℎ (c) Circuit to find 𝐼𝐿

Step 1 Determine the Thevenin equivalent voltage VTh.


For this, remove the load, i.e. the 8Ω resistance between A-B as shown in
Figure 2.24(a) and find VOC = VAB
12
VOC = VTh. = 18 
= 12V
6 + 12
Step 2 Determine the Thevenin’s equivalent resistance RTh
For this, remove RL = 8Ω , deactivate the source, i.e. short circuit the voltage
source and find the resistance seen looking in to the network from terminals
AB as shown in Figure 2.24(b).
𝑅𝑇ℎ = 4 + (12 ⃦6) = 4 + 4 = 8Ω
Step 3 Determination of I L
Knowing VTh. and RTh draw the Thevenin equivalent circuit as shown in Figure
2.24(c) connecting back RL and calculate I L using ohm’s law.
VTh. 12
IL = = = 0.75 A
RTh + RL 8 + 8
Verification by nodal analysis
The KCL equation at the only node present in the circuit of Figure 2.23 is

V1 − 18 V1 V
+ + 1 =0
6 12 4 + 8
 1 1 1  18
i.e.V1  + +  = =3
 6 12 12  6
Solving this node equation, we get the node voltage
3
𝑉1 = 1/3 = 9𝑉
V1 9V
Hence I1 = = = 0.75A
4 + 8 12
This shows the validity of Thevenin’s theorem

28
Example 2.12 Using Thevenin’s theorem, determine the current in the 2Ω
resistance of the network shown in Figure 2.25(a).

6 8 6 8 3.75
A
I I
12V
12V 6 4 2 6 I1 4 Voc
12V

B
6 (a) 10 6 10 6
(b) (c)

Figure 2.25(a) Circuit for Example 2.12 (b) Circuit with 2Ω resistance open
(c) Its equivalent to find I
Solution: Step I Determination of VTh
To determine the current in the 2Ω resistance of the network shown in Figure
2.25(a), we first open the 2Ω resistance as shown in Figure 2.25(b). Voc is the
voltage across the 4Ω resistance. The 4Ω resistance is in series with the 6Ω
resistance and this 4Ω + 6Ω = 10Ω resistance is in parallel with the 6Ω resistance
and this combination [(4 + 6) ⃦6]Ω = 3.75Ω is in series with the 6Ω resistance
when viewed from the 12V source.
12
 I= = 1.23A
3.75 + 6
6 6
I1 = I  = 1.23  = 0.461A
6+6+4 16
Therefore, the voltage drop across the 4Ω resistance
VTh = Voc = 4 I1 = 4  0.461 = 1.846V
Step II Determination of RTh
To find RTh , replace the voltage source by a short circuit. The circuit becomes as
shown in Figure 2.26(a). Looking into the circuit from open circuited terminals
𝑅𝑇ℎ = [[(6 ⃦6) + 6] ⃦4] + 8 + 10 = 20.77Ω
RTh
6 8
A 16.769
VTh 1.846V
6 4 RTh 2

B
6 10 (b)

Figure 2.26 (a) Circuit to find RTh (b) Thevenin equivalent


Step III Determination of the current I

29
To find the current I through the 2Ω resistance, the left of AB of Figure 2.26(a) is
replaced by Thevenin equivalent circuit as shown in Figure 2.26(b) and load is
connected.
𝑉𝑇ℎ 1.846
∴𝐼=𝑅 = 20.77+2 = 0.081A
𝑇ℎ +2
Example 2.13 Using Thevenin’s theorem find the current flowing through the 10Ω
resistor of the network shown in Figure 2.27(a).
12 12
8 6A 8 6A
A

16V 16V 4
4 10 Voc

B
(a) (b)

Figure 2.27 (a) Circuit for Example 2.13 (b) Circuit with 10Ω resistance open
Solution: Step I Determination of VTh
To find the current through the 10Ω resistance using Thevenin’s theorem, open
circuit the resistance 10Ω and find Voc across the open circuit. In Figure 2.27(b), with
AB open, the 6A current will flow through the 12Ω resistance causing a voltage drop
of 6 12 = 72V across it.
 VTh = Voc = 16V + ( 6 12 ) = 88V
Step II Determination of RTh
To find RTh , open circuit the current source and short circuit the voltage source as
shown in Figure 2.28(a). Now the (8Ω + 4Ω ) is shorted and RTh = 12 + 0 = 12Ω .
Step III Determination of current
The Thevenin equivalent circuit with load connected is shown in Figure 2.28(b). The
current through the resistance 10Ω is
88
I = VTh / (10 + RTh ) = = 4A
10 + 12

12
RTh
8
A
RTh VTh 10
4
(b)
B
(a)

Figure 2.28 (a) Circuit with sources deactivated (b) Thevenin equivalent
Example 2.14 Determine the current through the 8Ω resistance in the circuit shown
in Figure 2.29 using Thevenin’s theorem.

30
2 2 2 2

20V 4 4 20V I1 4 I2 4
6 6

8 6 6
A B
A B
(a) (b)

Figure 2.29 (a) Circuit for Example 2.14 (b) Circuit with AB open
Solution: Step I Determination of VTh
To find the current through the 8Ω resistance using Thevenin’s theorem, open circuit
the terminals AB and find the open circuit voltage Voc across AB.
Writing the mesh equations for the two loops in Figure 2.29(b), we get
2 I1 + 4 ( I1 − I 2 ) + 6 I1 − 20 = 0
i.e. 12 I1 − 4 I 2 = 20
and 2 I 2 + 4 I 2 + 6 I 2 + 4 ( I 2 − I1 ) = 0
i.e. −4 I1 + 16 I 2 = 0 or I1 = 4 I 2
 12 ( 4 I 2 ) − 4 I 2 = 20
20
i.e. 44 I 2 = 20 or I 2 == 0.4545A
44
and I1 = 4 I 2 = 4  0.4545 = 1.818A
 Voltage drop across the 6Ω resistance = Voc = VTh
VTh = 6 I 2 = 6  0.4545 = 2.727V
Step II Determination of RTh
To find RTh , short circuit the voltage source. The circuit appears as shown in Figure
2.30(a). Calculate the resistance looking into the circuit from the open circuited
terminals.
𝑅𝑇ℎ = [[(2 + 6) ⃦4] + 2 + 4] ⃦6
⃦ = 3.545Ω

2 2
RTh
4 4 A
3.545
VTh 2.727V 8
6 6
A B B
RTh
(a) (b)

Figure 2.30 (a) Circuit to find RTh (b) Thevenin equivalent circuit
Step III Determination of current I

31
The Thevenin equivalent circuit with load connected is shown in Figure 2.30(b). The
current through the 8Ω resistor I is
VTh 2.727
I= = = 0.236A
RTh + 8 3.545 + 8
2.9 NORTON’S THEOREM
This theorem is the converse of the Thevenin’s theorem. The Norton equivalent
circuit consists of an equivalent current source in parallel with the internal resistance
of the network.
Norton’s theorem states that any two-terminal linear active bilateral network
with resistances, voltage sources and/or current sources can be replaced by a
simple equivalent circuit consisting of a single current source in parallel with a single
resistance. The current source I sc being the short circuited current through the load
terminals and the resistance RN being the internal resistance of the source network
looking through the open circuited load terminals.
Norton’s theorem is a corollary to the Thevenin’s theorem.

Procedure to find the current through a branch using Norton’s theorem


1. Remove the branch through which current is to be found and mark the
terminals AB.
2. Short circuit the terminals AB and find the current through it and denote it as
I sc .
3. Replace the independent sources by their internal resistances. (If the internal
resistances are zero, then voltage sources are to be short circuited and
current sources are to be open circuited).
4. Calculate RTh ( RN ) between the terminals AB.
5. Connect the short circuit (Norton’s) current source I sc in parallel with RTh ( RN )
with output terminals AB.
6. Connect the removed branch between the terminals AB and find the current
RTh
through it using current divider formula IL = IN
RTh + RL
Illustration of Norton’s theorem with an example.

Consider a simple network shown in Figure 2.31

6 4
A

20V
8 12

Figure 2.31 (a) Circuit for explanation of Norton’s theorem

32
The current through the load resistance RL = 12Ω may be calculated using Norton’s
theorem as per the following steps.

6 4 6 4
A A

20V R N I N R N
8 I x I N
8
1.538A 7.429
12

B B
(a) (b) (c)

Figure 2.32 (a) circuit to find I sc (b) circuit to find RN (c) Norton’s circuit to find I L

Step I Calculation of I sc or I N

First remove the load resistance RL from the network terminals AB, short the
terminals AB as shown in Figure 2.32(a) and calculate the current I sc or I N through
the short circuit.

20 8
𝐼𝑆𝐶 = 𝐼𝑁 = × = 1.538A
6 + (4 ⃦8) 8 + 4

Step II Calculation of RN or RTh

Open the terminals AB, replace the voltage source by a short circuit and calculate
RTh , the resistance between AB, looking into the network from terminals AB as
shown in Figure 2.32(b).

⃦ ) = 7.429Ω
𝑅𝑁 = 𝑅𝑇ℎ = 4 + (6 8

Step III Calculation of current through load.


Draw the Norton’s equivalent circuit w.r.t. terminals AB as shown in Figure 2.32(c).

RN 7.429
IL = IN  = 1.538  0.588 A
RN + RL 7.429 + 12
Verification by Nodal analysis

For the given circuit shown in Figure 2.31, the KCL equation at the only node is

V1 − 20 V1 V1
+ + =0
6 8 16
 1 1 1  20
i.e. V1  + +  =
 6 8 16  6
Solving it, we get V1 = 9.415V .

So current through the load resistance of 12Ω is

33
V 9.415
IL =
= = 0.588 A
4 + 12 16
This shows the validity of Norton’s theorem

Example 2.15 Determine the current through the 10Ω resistance shown in Figure
2.33 using Norton’s theorem.
5
A
I
+
15 10
30V
-

Figure 2.33 Circuit for Example 2.15


Solution: Step I Determination of I sc
To determine I sc , short circuit the terminals AB and find the short circuit current I sc
as shown in Figure 2.34(a).
5
A A A
I

+ 15
I sc 5 15
RN 10
30V I sc RN
-

B B B
(a) (b) (c)
Figure 2.34 (a) Circuit to find I sc (b) Circuit to find RN (c) Norton equivalent to find I
30
From Figure 2.34(a) I sc = = 6A
5
Step II Determination of RN
5×15
From Figure 2.34(b) 𝑅𝑁 = 5 ⃦15 = 5+15 = 3.75Ω
Step III Determination of I
RN 3.75
From Figure 2.34(c) I = I sc = 6 = 1.636A
RN + 10 3.75 + 10
Example 2.16 Determine the current in the 12Ω resistance shown in Figure 2.35
using Norton’s theorem.

4 6

6
A

4 6
20A 12

10V
B

Figure 2.35 Circuit for Example 2.16


Solution: Step I Determination of I sc or I N

34
To apply Norton’s theorem, short circuit the output terminals AB as shown in Figure
2.36(a). Find 𝐼𝑠𝑐 using superposition theorem.
From Figure 2.36(b)
4
I sc1 = 20  = 5.714A
4+4+6
From Figure 2.36(c)
10
I sc 2 = = 0.714A
4+4+6
 Norton’s equivalent current source = I N = I sc = I sc1 + I sc 2 = 5.714 + 0.714 = 6.428A

4 6 4 6 4 6

6 6 6
A A

4 6 4 6 4 6
20A I sc 20A I sc1 I
10V 10V
B B
(a) (b) (c)

Figure 2.36 (a) Circuit with AB shorted (b) circuit with only 20A source
(c) Circuit with only 10V source
Step II Determination of RN or RTh
Norton’s equivalent resistance RN can be obtained from the circuit shown in Figure
2.37(a).

4 6
A
6
A 12
6.428A 4.2
4 6
RN
B
B (b)
(a)

Figure 2.37(a) Circuit to find 𝑅𝑁 (b) Norton’s equivalent circuit


From Figure 2.37(a)
𝑅𝑁 = 6 ⃦(6 + 4 + 4) = 4.2Ω
Step III Determination of current through the 12 Ω resister
The Norton’s equivalent circuit is shown in Figure 2.37(b). Current through 12Ω
resistance is
4.2
I = 6.428  = 1.666A
4.2 + 12
2.10 MAXIMUM POWER TRANSFER THEOREM
Electric circuits basically consist of sources supplying voltage, current or power to
the load. For many applications, it is desirable to transfer maximum amount of
power from an active network to an external load resistor RL . For example, in an

35
audio sound system, it is desirable to obtain maximum power transfer from the
output of the amplifier to the speaker. Maximum power transfer theorem gives a
condition for maximum power transfer from source to load.
The Maximum power transfer theorem states that a resistive load connected
to a dc network receives maximum power when the load resistance is equal to the
internal resistance of the source network as seen from the load terminals (i.e. the
Thevenin equivalent resistance RTh of the network).
The efficiency of the power transfer is 50%. So only communication circuits
are operated under the maximum power transfer condition but not the power circuits.
Procedure to be followed to apply maximum power transfer theorem
1. Remove the variable load resister RL from terminals AB.
2. Find the open circuit voltage VTh across points A and B.
3. Find the resistance RTh as seen from points A and B after replacing all sourceby
their internal resistances.
3. Find the resistance RL for maximum power transfer. RL = RTh
5. Find the maximum power transferred.
VTh V
IL = = Th
RTh + RL 2 RTh
VTh 2 VTh 2
Pmax = I RL =
2
L  RTh =
4 RTh 2 4 RTh
Example 2.17 Determine the maximum power delivered to the load in the circuit
shown in Figure 2.38
6 4 4
A

100V
+ 12 8 33.32V
+
- RL - RL

Figure 2.38 (a) Circuit for Example 2.17 (b) Thevenin equivalent circuit
Solution: To determine the maximum power delivered to the load, first we have to
determine the Thevenin equivalent circuit across the load as shown in Figure 2.39
6 4 6 4
A A

IT I
100V
+ 12 8 Voc 12 8 RTh
-
B B
(a) (b)

Figure 2.39 (a) Circuit to find Voc (b) Circuit to determine RTh
Step I Determination of VTh
From Figure 2.39(a), total resistance from source side is

36
𝑅𝑇 = [(8 + 4) ⃦12] + 6 = 12Ω
The current drawn by the circuit is
100 100
𝐼𝑇 = = = 8.33A
[(8 + 4) ⃦12] + 6 12

12 12
 I = IT  = 8.33  = 4.165A
12 + 4 + 8 24
 The current in the 8Ω resistor is 4.165A
 VTh = VOC = I  8 = 4.165  8 = 33.32V
Step II Determination of RTh
Thevenin resistance RTh is obtained as shown in Figure 2.39(b)
6 × 12
𝑅𝑇ℎ = [(6 ⃦12) + 4] ⃦8
⃦ = [( ) + 4] ⃦8
⃦ = 8 ⃦8
⃦ = 4Ω
6 + 12
Step III Determination of maximum power delivered to the load
The Thevenin equivalent circuit with load connected is shown in Figure 2.39(b)
From Figure 2.39(b) and maximum power transfer theorem RL = RTh = 4Ω
Therefore, current drawn by the load resistance
VTh 33.32
IL = = = 4.165A
RTh + RL 4 + 4
Power delivered to the load = I L2 RL = ( 3.165)  4 = 69.39W
2

( 33.32 ) = 69.39W
2
V2
Or Power delivered to the load = Th =
4 RL 4 4
Example 2.18 In the circuit shown in Figure 2.40(a), determine the load resistance
to receive the maximum power from the source. Also find the maximum power
delivered to the load.
12.67 A
10
RTh
8

RL +
80V A 6.67V
B - VTh RL
24
20

(a) (b)

Figure 2.40 (a) Circuit for Example 2.18 (b) Its Thevenin equivalent circuit
Solution: To determine RL for maximum power transfer and the maximum power
transferred, we have to first obtain the Thevenin equivalent circuit.
Step I Determination of VTh
Open circuiting the load RL , the circuit shown in Figure 2.41(a) results.

37
8 10

8 10

80V A B
A B 24 20
24 20

(b)
(a)

Figure 2.41(a) Circuit to find VTh (b) Circuit to find RTh


24
Voltage at point A is VA = 80 
= 60V
24 + 8
20
Voltage at point B is VB = 80  = 53.333V
20 + 10
 VAB = VTh = VA − VB = 60 − 53.333 = 6.67V
Step II Determination of RTh
The Thevenin equivalent resistance can be determined from the circuit shown in
Figure 2.41(b).
𝑅𝐴𝐵 = 𝑅𝑇ℎ = [(8 ⃦24) + (10 ⃦20)]Ω = [6 + 6.67]Ω = 12.67Ω
So, the value of RL for maximum power transfer is RL = RTh = 12.67Ω
Step III Determination of Pmax
The Thevenin equivalent circuit with RL connected is shown in Figure 2.41(b)
According to maximum power transfer theorem, for maximum power transfer
RL = RTh = 12.67Ω
Maximum power delivered to the Load
V2 6.67 2
PL = I L2 RL = Th = = 0.878W
4 RTh 4 12.67
Example 2.19 Find the value of RL for maximum power transfer and also the
maximum power in the circuit shown in Figure 2.42
4

2 2 RTh
V1 V2 A
1.09

4A
20V
4 VTh 13.09V RL
RL

B
(a) (b)

Figure 2.42 (a) Circuit for Example 2.19 (b) Its Thevenin equivalent
Solution: To find RL for maximum power transfer and the maximum power
transferred, we have to first obtain the Thevenin equivalent circuit w.r.t terminals AB.
Step I Determination of VTh
Open circuiting RL , we get the circuit shown in Figure 2.43(a)

38
4
4

V1 2 2 V2
A 2 2
V1 V2 A
1.5
A
4A 20V 4 RTh 4
4

B B
B
(a) (b) (c)

Figure 2.43 (a) Circuit with RL open (b) Circuit to find RTh
From Figure 2.43a), we have
V1 − 20 V1 − V2 V V V
+ + 4 = 0 , i.e. 1 + 1 − 2 = 10 − 4, i.e. 3V1 − V2 = 24
2 4 2 4 4
V2 − V1 V2 − 20 V2 V V V V
+ + = 0, i.e. − 1 + 2 + 2 + 2 = 10, i.e. −V1 + 4V2 = 40
4 2 4 4 4 2 4
Solving the above two equations involving V1 and V2 , we get
144
V2 = = 13.09V and V1 = 12.36V
11
 VTh = VOC = V2 = 13.09V
Step II Determination of RTh
To find RTh , open circuit the current source and short circuit the voltage source as
shown in Figure 2.43(b) and look into the circuit from the open circuited terminals.
𝑅𝑇ℎ = {[(2 + 4) ⃦2] ⃦4}Ω = 1.09Ω
Step III Determination of Pmax
The Thevenin equivalent circuit is shown in Figure 2.43(b)
For maximum power transfer, RL = RTh
The maximum power transferred is given by
VTh2 13.092
P= = = 39.3W
4 RL 4 1.09

SHORT QUESTIONS AND ANSWERS

34. Why superposition principle cannot be applied directly to find power?


A. Superposition principle cannot be applied directly to find power because current
and power are not linearly related.

35. How do you deactivate a voltage source/current source?


A. A voltage source/current source is deactivated by replacing the source with its
internal resistance.

36. How do you deactivate an ideal voltage source/current source?

39
A. An ideal voltage source is deactivated by replacing it with a short circuit and an
ideal current source is deactivated by replacing it with an open circuit.

37. State superposition theorem


A. Superposition theorem states that in a linear bilateral network containing two or
more independent sources, the voltage across or current through any branch is
equal to the algebraic sum of individual voltages or currents produced by each
independent source acting separately with all other independent sources set
equal to zero, i.e. replaced by their internal resistances. (An ideal voltage source
is to be replaced by a short circuit and an ideal current source is to be replaced
by an open circuit).

38. State Thevenin’s theorem


A. Thevenin’s theorem states that any linear active bilateral two terminal network
consisting of resistances and independent and/or dependant voltage and/or
current sources can be replaced by a simple equivalent circuit consisting of a
single voltage source 𝑉𝑇ℎ in series with a resistance 𝑅𝑇ℎ , where the value of the
voltage source 𝑉𝑇ℎ is equal to the open circuit voltage across the two terminals of
the network, and the resistance 𝑅𝑇ℎ is the equivalent resistance between the
terminals with all the energy sources replaced by their internal resistances .

39. If a circuit contains no independent energy sources, what are the values of
𝑉𝑇ℎ and 𝐼𝑁 ?
A. If a circuit contains no independent energy sources, both 𝑉𝑇ℎ and 𝐼𝑁 are zero and
the circuit has only the equivalent Thevenin resistance.

40. State Norton’s theorem


A. Norton’s theorem states that any linear active two terminal network with
resistances, independent and/or dependent voltage and/or current sources can
be replaced by a simple equivalent circuit consisting of a single current
source 𝐼𝑁 in parallel with resistance 𝑅𝑁 , where the value of the current
source 𝐼𝑁 is the short circuit current between the two terminals of the network
and the resistance 𝑅𝑁 is the equivalent resistance measured between the
terminals of the network with all the energy sources replaced by their internal
resistances. This is the converse of the Thevenin’s theorem.

41. State maximum power transfer theorem.


A. The maximum power transfer theorem states that a resistive load connected to a
dc network receives maximum power when the load resistance is equal to the
internal resistance of the source network as seen from the load terminals. or
The maximum power transfer theorem states that maximum power is
delivered from a source to a load when the load resistance is equal to the source
resistance.

40
42. What is the efficiency of the circuit under the condition of maximum power
transfer?
A. Under maximum power transfer condition, the efficiency of the circuit is 50%.

PROBLEMS
2.6P Find the currents in the different resistances of the circuit shown in Figure
2.6P using superposition theorem.
20Ω 30Ω
R1 R2

32V R3 20Ω 20V

Figure 2.6P

2.7P Using superposition theorem, find the voltage across the 5Ω resistance in the
circuit of Figure 2.7P.

10Ω

A
4A

2A 5Ω 2Ω

B
Figure 2.7P

2.8P Calculate the current in the 2Ω resistor of Figure 2.8P using Superposition
theorem.

a

+
2V 1Ω 1A 2Ω
-

b
Figure 2.8P

2.9P Calculate the current in the 2Ω resistor of Figure 2.9P using Thevenin’s
theorem.

41

a

+
2V 1Ω 1A 2Ω
-

Figure 2.9P

2.10P In the network shown in Figure 2.10P, find the current through the 5Ω resistor
using Thevenin’s theorem.
2Ω 4Ω


20V 5Ω 12V
10V

Figure 2.10P

2.11P Determine the Thevenin and Norton equivalent circuits w.r.t terminals ab for
the circuit shown in Figure 2.11P.
5A
a

12A 12Ω 4Ω 3A

b
Figure 2.11P

2.12P Determine the Thevenin and Norton equivalent circuit for the given circuit
shown in Figure 2.12P.

3Ω 4Ω

A

25A 2Ω 5Ω

Figure 2.12P

2.13P Find the value of RL to be connected between the terminals AB of Figure


2.13P for maximum power transfer. Also find the power transferred.

42
2Ω 3Ω
A


3A 2Ω RL
-
5V

+
B

Figure 2.13P

2.14P Find the value of RL to be connected between the terminals AB of Figure


2.14P for maximum power transfer. Also find the power transferred.


3A Ω 6Ω
30
50
Ω

20V
b

Figure 2.14P

43

You might also like